assumption question

This topic has expert replies
Legendary Member
Posts: 995
Joined: Tue Apr 13, 2010 11:56 pm
Thanked: 31 times
Followed by:1 members

assumption question

by paes » Sat Jun 05, 2010 10:28 pm
Politician: Since my administration entered office, the percentage of people looking for work but unable to find it
has dropped from 7% to 5% of the total number of people either working or looking for work. These statistics
show that my administration's business-friendly policies have helped reduce the level of unemployment among
those willing to work.
Which of the following is an assumption on which the politician's argument relies?
A. The previous administration's worker re-education program had no significant effect on the unemployment
rate.
B. Less than 2% of the population working or willing to work at the start of the politician's term have stopped
working or looking for work.
C. None of the new jobs created during the politician's term went to those who already had jobs.
D. Most of those looking for employment found jobs in the field they were trained in.
E. The politician's business-friendly policies have had a direct affect on the ability of firms to hire more workers.

OA after some discussions.

Master | Next Rank: 500 Posts
Posts: 268
Joined: Wed Mar 17, 2010 2:32 am
Thanked: 17 times

by this_time_i_will » Sat Jun 05, 2010 10:35 pm
Should be C. Otherwise the politician can not claim the reduction in unemployment.

User avatar
Legendary Member
Posts: 535
Joined: Fri Jun 08, 2007 2:12 am
Thanked: 87 times
Followed by:5 members
GMAT Score:730

by hardik.jadeja » Sat Jun 05, 2010 10:47 pm
Clearly the answer is B.

If the reduction in the percent of people who are looking for job is because some of them have stopped working or looking for work, then politicians claim that his administration's business-friendly policies have helped reduce the level of unemployment among those willing to work is wrong.

Master | Next Rank: 500 Posts
Posts: 305
Joined: Mon Jul 27, 2009 5:38 am
Thanked: 10 times

by Shawshank » Sat Jun 05, 2010 11:04 pm
For me the fight is betwen B and C
If you negate both statements u will see that C fails the negation test.

Hence IMO -- B
++++++++++++++++++++++++++++++
Shawshank Redemtion -- Hope is still alive ...

Master | Next Rank: 500 Posts
Posts: 134
Joined: Mon Mar 01, 2010 7:08 am
Thanked: 3 times

by bupbebeo » Sun Jun 06, 2010 12:14 am
paes wrote:Politician: Since my administration entered office, the percentage of people looking for work but unable to find it
has dropped from 7% to 5% of the total number of people either working or looking for work. These statistics
show that my administration's business-friendly policies have helped reduce the level of unemployment among
those willing to work.
Which of the following is an assumption on which the politician's argument relies?
A. The previous administration's worker re-education program had no significant effect on the unemployment
rate.
B. Less than 2% of the population working or willing to work at the start of the politician's term have stopped
working or looking for work.
C. None of the new jobs created during the politician's term went to those who already had jobs.
D. Most of those looking for employment found jobs in the field they were trained in.
E. The politician's business-friendly policies have had a direct affect on the ability of firms to hire more workers.

OA after some discussions.
I strongly believe the correct answer is B.

If > 2% population working and willing to work have stopped working and looking at the start of the politician's term, the reduction has nothing to do with his policy. In other words, it weakens the argument. when we negate the answer choice, then it weaken the arguments. Then, it should be the correct answer for the question.

Hope that helps

Master | Next Rank: 500 Posts
Posts: 226
Joined: Wed Jun 02, 2010 1:46 am
Thanked: 2 times

by martin.jonson007 » Sun Jun 06, 2010 4:23 am
IMO B

Newbie | Next Rank: 10 Posts
Posts: 6
Joined: Sun May 02, 2010 1:08 pm

by bmehra » Sun Jun 06, 2010 12:57 pm
Should be B. Please do post the OA.

Legendary Member
Posts: 995
Joined: Tue Apr 13, 2010 11:56 pm
Thanked: 31 times
Followed by:1 members

by paes » Sun Jun 06, 2010 5:04 pm
OA is B.

User avatar
Legendary Member
Posts: 537
Joined: Sun Jul 19, 2009 7:15 am
Location: Nagpur , India
Thanked: 41 times
Followed by:1 members

by rockeyb » Sun Jun 06, 2010 9:20 pm
paes wrote:OA is B.
Hey can you please tell me the source of this question , because I dont think the correct answer should be B .

Reason : Option B states that

Less than 2% of the population working or willing to work at the start of the politician’s term have stopped
working or looking for work.

If this was true then it weakens the mayor's conclusion that states that its was because of his policies that there was a reduction in jobs .


Infact the only options that bridges the gap between the conclusion and premis is E that sates that mayor's policies allowed firms to hire more people and created more jobs .

I hope this makes sense . I may be wrong .

Any addition thoughts are welcome .
"Know thyself" and "Nothing in excess"

GMAT Instructor
Posts: 1302
Joined: Mon Oct 19, 2009 2:13 pm
Location: Toronto
Thanked: 539 times
Followed by:164 members
GMAT Score:800

by Testluv » Sun Jun 06, 2010 9:38 pm
Hi rockeyb,

choice B is in fact correct.

The politician argues that the cause of the reduced fraction of people looking for work is something good about his administration. But it could be because people simply got frustrated and stopped looking for work. The author is assuming that this is not the cause. Because the percentage went from 7 to 5, the author is assuming that <2% stopped looking for work. If 2% or >2% of people stopped looking for work, then the politician's argument is defeated as it would indicate that the administration's business-friendly policies was not the cause of the reduced percentage; instead, the cause would be people losing hope of getting a job.
Kaplan Teacher in Toronto

User avatar
Legendary Member
Posts: 537
Joined: Sun Jul 19, 2009 7:15 am
Location: Nagpur , India
Thanked: 41 times
Followed by:1 members

by rockeyb » Sun Jun 06, 2010 9:51 pm
Testluv wrote:Hi rockeyb,

choice B is in fact correct.

The politician argues that the cause of the reduced fraction of people looking for work is something good about his administration. But it could be because people simply got frustrated and stopped looking for work. The author is assuming that this is not the cause. Because the percentage went from 7 to 5, the author is assuming that <2% stopped looking for work. If 2% or >2% of people stopped looking for work, then the politician's argument is defeated as it would indicate that the administration's business-friendly policies was not the cause of the reduced percentage; instead, the cause would be people losing hope of getting a job.
Hey Testluv ,

Thanks for your reply .

I got the point you are trying to make here . The difference is between less than 2% and 2% or more than 2% .

Phew!!! need to read carefully .

Thanks again .
"Know thyself" and "Nothing in excess"

Legendary Member
Posts: 995
Joined: Tue Apr 13, 2010 11:56 pm
Thanked: 31 times
Followed by:1 members

by paes » Sun Jun 06, 2010 10:32 pm
thanks testluv.

You have given a perfect explanation.

I also selected E initially.

Source : 800score

GMAT Instructor
Posts: 1302
Joined: Mon Oct 19, 2009 2:13 pm
Location: Toronto
Thanked: 539 times
Followed by:164 members
GMAT Score:800

by Testluv » Sun Jun 06, 2010 11:02 pm
paes wrote:thanks testluv.

You have given a perfect explanation.

I also selected E initially.

Source : 800score
Thanks. I can see why choice E is tempting. But he doesn't have to assume that there was a "direct" effect.
Kaplan Teacher in Toronto

Newbie | Next Rank: 10 Posts
Posts: 8
Joined: Sun May 30, 2010 12:15 am

by deepaks04 » Mon Jun 07, 2010 12:07 am
Hi TestLuv,

If politician will be making an assumption which will strengthen his argument which in case should be 'E', whereas B negates his argument...

Can you please explain it further.

Master | Next Rank: 500 Posts
Posts: 292
Joined: Fri Jul 17, 2009 8:39 am
Thanked: 6 times
Followed by:1 members

by pnk » Mon Jun 07, 2010 10:50 am
Hi TestLuv,

Cud you explain why E is incorrect - is it bcoz of firms to hire more workers (numbers) or something else.

thx